Stay ahead of learning milestones! Enroll in a class over the summer!

G
Topic
First Poster
Last Poster
Estimate on number of progressions
Assassino9931   0
an hour ago
Source: RMM Shortlist 2024 C4
Let $n$ be a positive integer. For a set $S$ of $n$ real numbers, let $f(S)$ denote the number of increasing arithmetic progressions of length at least two all of whose terms are in $S$. Prove that, if $S$ is a set of $n$ real numbers, then
\[ f(S) \leq \frac{n^2}{4} + f(\{1,2,\ldots,n\})\]
0 replies
Assassino9931
an hour ago
0 replies
Popular children at camp with algebra and geometry
Assassino9931   0
an hour ago
Source: RMM Shortlist 2024 C3
Fix an odd integer $n\geq 3$. At a maths camp, there are $n^2$ children, each of whom selects
either algebra or geometry as their favourite topic. At lunch, they sit at $n$ tables, with $n$ children
on each table, and start talking about mathematics. A child is said to be popular if their favourite
topic has a majority at their table. For dinner, the students again sit at $n$ tables, with $n$ children
on each table, such that no two children share a table at both lunch and dinner. Determine the
minimal number of young mathematicians who are popular at both mealtimes. (The minimum is across all sets of topic preferences and seating arrangements.)
0 replies
Assassino9931
an hour ago
0 replies
Triangles in dissections
Assassino9931   0
an hour ago
Source: RMM Shortlist 2024 C2
Fix an integer $n\geq 3$ and let $A_1A_2\ldots A_n$ be a convex polygon in the plane. Let $\mathcal{M}$ be the set of all midpoints $M_{i,j}$ of segments $A_iA_j$ where $i\neq j$. Assume that all of these midpoints are distinct, i.e. $\mathcal{M}$ consists of $\frac{n(n-1)}{2}$ elements. Dissect the polygon $M_{1,2}M_{2,3}\ldots M_{n,1}$ into triangles so that the following hold:

(1) The intersection of every two triangles (interior and boundary) is either empty or a common
vertex or a common side.
(2) The vertices of all triangles lie in M (not all points in M are necessarily used).
(3) Each side of every triangle is of the form $M_{i,j}M_{i,k}$ for some pairwise distinct indices $i,j,k$.

Prove that the total number of triangles in such a dissection is $3n-8$.
0 replies
Assassino9931
an hour ago
0 replies
IMO Shortlist Problems
ABCD1728   1
N an hour ago by mrtheory
Source: IMO official website
Where can I get the official solution for ISL before 2005? The official website only has solutions after 2006. Thanks :)
1 reply
ABCD1728
Yesterday at 12:44 PM
mrtheory
an hour ago
Tangency geo
Assassino9931   0
an hour ago
Source: RMM Shortlist 2024 G1
Let $ABC$ be an acute triangle with $\angle ABC > 45^{\circ}$ and $\angle ACB > 45^{\circ}$. Let $M$ be the midpoint of the side $BC$. The circumcircle of triangle $ABM$ intersects the side $AC$ again at $X\neq A$ and the circumcircle of triangle $ACM$ intersects the side $AB$ again at $Y\neq A$. The point $P$ lies on the perpendicular bisector of the segment $BC$ so that the points $P$ and $A$ lie on the same side of $XY$ and $\angle XPY = 90^{\circ} + \angle BAC$. Prove that the circumcircles of triangles $BPY$ and $CPX$ are tangent.
0 replies
1 viewing
Assassino9931
an hour ago
0 replies
Inequalities in real math research
Assassino9931   0
an hour ago
Source: RMM Shortlist 2024 A3
For a positive integer $n$ denote $F_n(x_1,x_2,\ldots,x_n) = 1 + x_1 + x_1x_2 + \cdots +x_1x_2\ldots x_n$. For any real numbers $x_1\geq x_2 \geq \ldots \geq x_k \geq 0$ prove that
\[ \prod_{i=1}^k F_i(x_{k-i+1},x_{k-i+2},\ldots,x_k) \geq \prod_{i=1}^k F_i(x_i,x_i,\ldots,x_i)\]
0 replies
1 viewing
Assassino9931
an hour ago
0 replies
Two circles, a tangent line and a parallel
Valentin Vornicu   104
N 2 hours ago by cubres
Source: IMO 2000, Problem 1, IMO Shortlist 2000, G2
Two circles $ G_1$ and $ G_2$ intersect at two points $ M$ and $ N$. Let $ AB$ be the line tangent to these circles at $ A$ and $ B$, respectively, so that $ M$ lies closer to $ AB$ than $ N$. Let $ CD$ be the line parallel to $ AB$ and passing through the point $ M$, with $ C$ on $ G_1$ and $ D$ on $ G_2$. Lines $ AC$ and $ BD$ meet at $ E$; lines $ AN$ and $ CD$ meet at $ P$; lines $ BN$ and $ CD$ meet at $ Q$. Show that $ EP = EQ$.
104 replies
Valentin Vornicu
Oct 24, 2005
cubres
2 hours ago
Austrian Regional MO 2025 P4
BR1F1SZ   2
N 3 hours ago by NumberzAndStuff
Source: Austrian Regional MO
Let $z$ be a positive integer that is not divisible by $8$. Furthermore, let $n \geqslant 2$ be a positive integer. Prove that none of the numbers of the form $z^n + z + 1$ is a square number.

(Walther Janous)
2 replies
BR1F1SZ
Apr 18, 2025
NumberzAndStuff
3 hours ago
Austrian Regional MO 2025 P3
BR1F1SZ   1
N 3 hours ago by NumberzAndStuff
Source: Austrian Regional MO
There are $6$ different bus lines in a city, each stopping at exactly $5$ stations and running in both directions. Nevertheless, for every two different stations there is always a bus line connecting these two stations. Determine the maximum number of stations in this city.

(Karl Czakler)
1 reply
BR1F1SZ
Apr 18, 2025
NumberzAndStuff
3 hours ago
Austrian Regional MO 2025 P2
BR1F1SZ   2
N 3 hours ago by NumberzAndStuff
Source: Austrian Regional MO
Let $\triangle{ABC}$ be an isosceles triangle with $AC = BC$ and circumcircle $\omega$. The line through $B$ perpendicular to $BC$ is denoted by $\ell$. Furthermore, let $M$ be any point on $\ell$. The circle $\gamma$ with center $M$ and radius $BM$ intersects $AB$ once more at point $P$ and the circumcircle $\omega$ once more at point $Q$. Prove that the points $P,Q$ and $C$ lie on a straight line.

(Karl Czakler)
2 replies
BR1F1SZ
Apr 18, 2025
NumberzAndStuff
3 hours ago
n-gon function
ehsan2004   10
N Apr 4, 2025 by Zany9998
Source: Romanian IMO Team Selection Test TST 1996, problem 1
Let $ f: \mathbb{R}^2 \rightarrow \mathbb{R} $ be a function such that for every regular $ n $-gon $ A_1A_2 \ldots A_n $ we have $ f(A_1)+f(A_2)+\cdots +f(A_n)=0 $. Prove that $ f(x)=0 $ for all reals $ x $.
10 replies
ehsan2004
Sep 13, 2005
Zany9998
Apr 4, 2025
n-gon function
G H J
G H BBookmark kLocked kLocked NReply
Source: Romanian IMO Team Selection Test TST 1996, problem 1
The post below has been deleted. Click to close.
This post has been deleted. Click here to see post.
ehsan2004
2238 posts
#1 • 3 Y
Y by Adventure10, centslordm, Mango247
Let $ f: \mathbb{R}^2 \rightarrow \mathbb{R} $ be a function such that for every regular $ n $-gon $ A_1A_2 \ldots A_n $ we have $ f(A_1)+f(A_2)+\cdots +f(A_n)=0 $. Prove that $ f(x)=0 $ for all reals $ x $.
Z K Y
The post below has been deleted. Click to close.
This post has been deleted. Click here to see post.
perfect_radio
2607 posts
#2 • 2 Y
Y by Adventure10, Mango247
ehsan2004 wrote:
Prove that $f(x)=0$ for all reals $x$.

You wanted to say "$f(A)=0$ for all points $A$", right?
Z K Y
The post below has been deleted. Click to close.
This post has been deleted. Click here to see post.
ehsan2004
2238 posts
#3 • 2 Y
Y by Adventure10, Mango247
perfect_radio wrote:
ehsan2004 wrote:
Prove that $f(x)=0$ for all reals $x$.

You wanted to say "$f(A)=0$ for all points $A$", right?

excuse me, my meant was $f(x)\equiv 0$
Z K Y
The post below has been deleted. Click to close.
This post has been deleted. Click here to see post.
perfect_radio
2607 posts
#4 • 2 Y
Y by Adventure10, Mango247
Take $A \neq B$. Let $\ell$ be the perpendicular bisector of $AB$. Construct a rhombus $ACBD$, with $C,D \in \ell$ and $\measuredangle DAC = \measuredangle DBC = \dfrac{\pi}{3}$. This yields $f(A)+f(C)+f(D)=0=f(B)+f(C)+f(D)$, so $f(A)=f(B)=t$, $\forall A \neq B$.

Therefore $nt=0$, so $t=0$.

Have I done something wrong? It looks too good to be true :blush: (because I used the condition given only for $n=3$)
Z K Y
The post below has been deleted. Click to close.
This post has been deleted. Click here to see post.
enescu
741 posts
#5 • 2 Y
Y by Adventure10, Mango247
Actually, $n$ is fixed in the original statement.
Z K Y
The post below has been deleted. Click to close.
This post has been deleted. Click here to see post.
perfect_radio
2607 posts
#6 • 2 Y
Y by Adventure10, Mango247
enescu wrote:
Actually, $n$ is fixed in the original statement.
Oops... sorry :( . do you know the solution for $n \geq 4$?
Z K Y
The post below has been deleted. Click to close.
This post has been deleted. Click here to see post.
enescu
741 posts
#7 • 7 Y
Y by Batominovski, wateringanddrowned, Adventure10, Upwgs_2008, Mango247, and 2 other users
Yes. Let $A$ be an arbitrary point. Consider a regular $n-$gon $AA_{1}A_{2}\ldots A_{n-1}.$ Let $k$ be an integer, $0\leq k\leq n-1.$ A rotation with center $A$ of angle $\dfrac{2k\pi}{n}$ sends the polygon $AA_{1}A_{2}\ldots A_{n-1}$ to $A_{k0}A_{k1}\ldots A_{k,n-1},$ where $A_{k0}=A$ and $A_{ki}$ is the image of $A_{i}$, for all $i=1,2,\ldots,n-1.$

From the condition of the statement, we have
\[ \sum_{k=0}^{n-1} \sum_{i=0}^{n-1}{f(A_{ki})}=0.  \]
Observe that in the sum the number $f(A)$ appears $n$ times, therefore
\[ nf(A)+\sum_{k=0}^{n-1} \sum_{i=1}^{n-1}{f(A_{ki})}=0.  \]
On the other hand, we have
\[ \sum_{k=0}^{n-1} \sum_{i=1}^{n-1}{f(A_{ki})}=\sum_{i=1}^{n-1} \sum_{k=0}^{n-1}{f(A_{ki})}=0,  \]
since the polygons $A_{0i}A_{1i}\ldots A_{n-1,i}$ are all regular $n-$gons. From the two equalities above we deduce $f(A)=0,$ hence $f$ is the zero function.
Z K Y
The post below has been deleted. Click to close.
This post has been deleted. Click here to see post.
TheUltimate123
1740 posts
#8 • 1 Y
Y by MS_asdfgzxcvb
We may assume \(n\) is even, since for \(n\) odd, the sum of the vertices in any \(2n\)-gon is zero.

Now let \(A_1\cdots A_n\) be a regular \(n\)-gon. For each \(i\) and \(j\), let \(M_{ij}\) be the midpoint of \(\overline{A_iA_j}\) (so in particular, \(M_iM_i=A_i\)), and let \(O\) be the center of the \(n\)-gon.

We know since \(M_{i1}M_{i2}\cdots M_{in}\) and \(M_{1,1+i}M_{2,2+i}\cdots M_{n,n+i}\) are regular \(n\)-gons that \begin{align*}     0=\sum_i\sum_jf(M_{ij})     =n\cdot f(O)+\sum_j\sum_{\substack{i<n\\ i\ne n/2}}f(M_{j,j+i})     &=n\cdot f(O) \end{align*}
This post has been edited 1 time. Last edited by TheUltimate123, Jun 30, 2021, 8:40 AM
Z K Y
The post below has been deleted. Click to close.
This post has been deleted. Click here to see post.
jasperE3
11288 posts
#9
Y by
ehsan2004 wrote:
Let $ f: \mathbb{R}^2 \rightarrow \mathbb{R} $ be a function such that for every regular $ n $-gon $ A_1A_2 \ldots A_n $ we have $ f(A_1)+f(A_2)+\cdots +f(A_n)=0 $. Prove that $ f(x)=0 $ for all reals $ x $.

The claim for just $n=4$:
https://aops.com/community/p1703551
Z K Y
The post below has been deleted. Click to close.
This post has been deleted. Click here to see post.
AshAuktober
1000 posts
#10
Y by
For $n$ even, draw a lot of $n$-gons with diametres the segments through the respective vertices and the centre of some $n$-gon, and the calculation works out to give $f(\text{ centre }) = 0$, so we're done.
For $n$ odd, notice that the statement then holds for $2n$ as well, so we're done.
Z K Y
The post below has been deleted. Click to close.
This post has been deleted. Click here to see post.
Zany9998
11 posts
#11
Y by
I wonder if there exists a coloring proof. i.e. label all negative points red, all 0 points yellow and all positive points green. I’ve proved that both red and green are dense in R^2 if whole board is not yellow. Is this sufficient to prove that there exists a regular n-gon whose vertices have at least one red and no green or vice versa? The condition seems to be strong enough.
Z K Y
N Quick Reply
G
H
=
a